Finding Laurent Series of 1/sinh(z) Up to z^5 Term

Click For Summary
The discussion focuses on finding the Laurent series of 1/sinh(z) up to the z^5 term, with the derived series being 1/z - z/3 + (7z^3/360) - (31z^5/15120). The user seeks confirmation on the correctness of their solution and raises issues with LaTeX code not updating in the forum preview. Suggestions for verifying the solution include checking values near zero and multiplying the Taylor series for sinh(x) with the Laurent series. There is also a mention of a forum bug causing old images to appear, with advice to refresh the preview page. The conversation highlights both mathematical inquiry and technical challenges in posting.
jjangub
Messages
20
Reaction score
0

Homework Statement


Find the Laurent series about 0 of 1/sinh up to (and including 0) the z5 term

Homework Equations


The Attempt at a Solution


Since 1/sinh is equal to
(1/z) * (1/(1+(z^2/3!)+(z^4/5!)+(z^6/7!)+...))
So if we work on the second term by dividing 1 by denominator and multiply by 1/z, we get
1/sinhz = 1/z -z/3 + (7z^3/360) - (31z^5/15120)
z is between 0 and pi
This is the answer I got.
Please tell me if I did something wrong.
P.S : And I tried to use LaTeX code for this post over an hour, but I failed.
Why is the old code keep appearing on the screen even though I replaced it with the new code when I see preview post?
Should I post it with LaTex code all the time?
Thank you.
 
Physics news on Phys.org
jjangub said:
So if we work on the second term by dividing 1 by denominator and multiply by 1/z
This sounds like a good approach.

z is between 0 and pi
Isn't it valid for all complex z? Anyways, that's a tangential issue.


If you want to make sure you didn't make an arithmetic error, I can think off the top of my head two reasonable ways to check your work:
  1. Plug a value near zero into both 1/sinh(x) and your Laurent series, and see if they are similar
  2. Multiply the Taylor series for sinh(x) and your Laurent series, to see if you get 1 (up to the appropriate order)
 
jjangub said:
P.S : And I tried to use LaTeX code for this post over an hour, but I failed.
Why is the old code keep appearing on the screen even though I replaced it with the new code when I see preview post?
Should I post it with LaTex code all the time?
Thank you.
There's a bug in the forum that causes previously cached images to appear instead of the revised images. If you refresh the preview page, it should load the new images.
 
Question: A clock's minute hand has length 4 and its hour hand has length 3. What is the distance between the tips at the moment when it is increasing most rapidly?(Putnam Exam Question) Answer: Making assumption that both the hands moves at constant angular velocities, the answer is ## \sqrt{7} .## But don't you think this assumption is somewhat doubtful and wrong?

Similar threads

  • · Replies 7 ·
Replies
7
Views
1K
  • · Replies 3 ·
Replies
3
Views
1K
  • · Replies 10 ·
Replies
10
Views
2K
  • · Replies 9 ·
Replies
9
Views
2K
  • · Replies 2 ·
Replies
2
Views
3K
  • · Replies 3 ·
Replies
3
Views
2K
  • · Replies 3 ·
Replies
3
Views
1K
  • · Replies 1 ·
Replies
1
Views
2K
  • · Replies 12 ·
Replies
12
Views
5K
Replies
2
Views
1K